Nehmen Sie die Kontinuumsgrenze von U(N)U(N)U(N)-Eichtheorien

Ich möchte Ihre Aufmerksamkeit auf den Anhang lenken C auf Seite 38 dieser Abhandlung .

  • Die gleichung C .2 es scheint die Summe auszuwerten R χ R ( U M ) in Gleichung 3.16 dieses Papiers . ich verstehe das U U ( N C ) Gruppe und dann mit N F (fundamentale?) Hypermultiplets Es scheint, dass anscheinend Folgendes behauptet wird, um wahr zu sein,

R χ R ( U M ) = 2 N F [ T R U M + T R U M ]

Kann jemand helfen, das obige abzuleiten?

  • Die erste Zeile der Gleichung C .4 scheint das gleiche zu sein wie gleichung 5.3 dieses früher verlinkten Papiers , aber nicht ganz ...

  • Ich war ziemlich verwirrt mit der Idee in der Gleichung C .5 Und C .6 die Eigenwerte der zu denken U ( N C ) Gruppe auf dem Kreis, der durch eine Verteilungsfunktion gegeben werden soll ρ ( θ ) 0 mit einer mysteriösen Normalisierung wie in der Gleichung C .5 .

    Ich würde mich freuen, wenn jemand helfen kann, zu verstehen, wie Gleichung C.6 bestimmt wurde. Hier scheinen viele Schritte übersprungen worden zu sein, die ich nur schwer wiederherstellen konnte!

  • Nach diesem Papier scheint es, dass die ursprüngliche allgemeinere Gleichung 2.43 auf Seite 17 hat zwei mögliche Grenzen wie in Gleichung 2.45 auf Seite 17, wenn es welche gibt C Materiefelder im Adjunkten und N C oder Gleichung 2.48 in der Veneziano-Grenze mit fundamentalen Materiefeldern.

    Ich würde gerne die Herleitung/Referenz für die oben zitierten Gleichungen 2.45 und 2.48 wissen.

    Ich denke, dies ist eines der lehrreichsten Mittel, um das Veneziano-Limit zu erreichen, in dem AdS/CFT funktionieren soll?

Antworten (1)

Der Charakter χ R : G C einer Vertretung R ist definiert durch χ R ( U ) = T R R ( U ) , nämlich durch die Spur von U in der Vertretung R . Siehe zum Beispiel Anhang A von Aharony et al. . Dann erscheint die Gleichung, die Sie schreiben, vernünftig: Vermutlich gibt es sie N F Hypermultiplets, jedes mit fundamentalen und antifundamentalen Feldern, die geben T R ( U M ) Und T R ( U M ) jeweils in der Summe über R . Die Spur wird von der Matrix genommen U weil die Felder in der Grundwelle liegen.

Der Faktor 2 sollte sich aus den Details ergeben, über welche Darstellungen genau summiert wird, dem Feldinhalt der Hypermultiplets usw., denen ich nicht zu folgen versucht habe.

Lassen Sie mich nun zur Verteilung der Eigenwerte, die in (C.5) erscheint, eine physikalische Erklärung geben, woher dies kommt. Sie betrachten die Thermodynamik einer Theorie über eine Kugel, also ist die Topologie die einer Kugel mal S 1 -- die euklidische Kompaktzeit. In diesem Fall gibt es einen Nullmodus des Eichfelds, der dadurch entsteht, dass man ein konstantes Feld in thermischer Richtung einschalten kann. Dieser Modus kann (im Allgemeinen) nicht durch eine Eichtransformation entfernt werden, daher müssen Sie in einem Pfadintegral darüber integrieren. Dies wird beispielsweise in Abschnitt 4.1 von Aharony et al. erklärt . Sie können eine Eichtransformation verwenden, um diese Nullmodusmatrix zu diagonalisieren, sodass Sie mit den diskreten Eigenwerten der Matrix zum Integrieren übrig bleiben. Sie können auch zeigen, dass diese Eigenwerte auf einem Kreis leben, weil Sie sie verschieben können 2 π (bei einigen Normalisierungen) mithilfe einer Messtransformation, sodass Sie sie über einen Kreis integrieren müssen. Im großen N Grenze haben Sie eine unendliche Anzahl solcher Eigenwerte, aber sie sind immer noch darauf beschränkt, auf einem Kreis zu leben. Sie müssen sie also mit einer Dichte beschreiben, und das nennen sie die Verteilung ρ .

Bearbeiten: Erklärung von (C.6) hinzufügen

Um (C.6) herzuleiten, betrachten wir zunächst die Summe

ich cos ( N a ich ) = N 0 2 π ρ ( θ ) cos ( N θ )

Wo ρ ( θ ) = 1 N ich δ ( θ a ich ) . Beachte das ρ hat die richtige Normierung.

Bei der Einnahme N das Delta funktioniert in ρ wird sehr dicht, und wir können uns annähern ρ durch eine glatte Funktion, deren Wert bei θ hängt von der Dichte der Deltafunktionen über das kleine Intervall ab [ θ , θ + ϵ ] . Dies ist eine gute Annäherung, da die Funktion cos ( N θ ) über die wir integrieren, wird in diesen Intervallen nicht viel variieren, also wird es das Ergebnis nicht viel ändern, wenn man es mittelt, anstatt es zu sampeln. Wenn wir ans Limit kommen N = Dies ist keine Annäherung mehr, da die Deltafunktionen stetig werden.

Das erklärt also die Herleitung des zweiten Terms in (C.6). Was den ersten Term betrifft, ist die Idee ähnlich, außer dass Sie zwei Integrale darüber haben θ , θ ' Entsprechend der ich , J Summen. Die Beiträge aus der ich = J Bedingungen ( θ = θ ' im Integral) sind im Großen subführend N Grenze: Sie skalieren als N während der Rest wie skaliert N 2 , sie werden also vernachlässigt.

Jetzt ersetzen cos ( N ( θ θ ' ) ) = cos ( N θ ) cos ( N θ ' ) + Sünde ( N θ ) Sünde ( N θ ' ) . Beachten Sie, dass im Pfad integral über ρ Sie können nur Eigenwertverteilungen berücksichtigen, die unter symmetrisch sind θ θ , weil der ursprüngliche Integrand darunter invariant ist. Dies wird zum Beispiel bei Schnitzer explizit erwähnt , der die gleiche Berechnung durchführt. Das bedeutet also D θ ρ Sünde ( N θ ) = 0 . Ich denke, ab diesem Punkt sollte es klar sein.

@ Guy Gur-Ari Jeder der N F Hypermultiplets hat, glaube ich, 2 N = 2 chirale Multipletts, eines in der adjungierten und das andere in der konjugierten Adjungierten (=adjungiert) der Eichgruppe. Jetzt haben wir χ A D J = χ A D J ¯ = χ F u N D × χ A N T ich F u N D . Also sollte jedes Hypermultiplet a beigetragen haben 2 χ F u N D × χ A N T ich F u N D ..Rechts?
Ich kann dieses allgemeine Argument sehen, dass der Eigenwert von a beschrieben werden muss U ( N ) Matrix durch eine Dichte auf dem Kreis, aber es ist die spezifische Umschreibung dessen wie in Gleichung C.6, die mich verwirrt. Es wäre eine große Hilfe, wenn Sie in ein paar Zeilen schreiben könnten, wie diese Gleichung zustande kam!
Die Materie-Multiplets, die sie in Anhang C betrachten, sind im Wesentlichen, wie sie am Anfang von Abschnitt 3.2 schreiben (kurz bevor sie sich auf den Anhang beziehen). Ich habe der Antwort die Erklärung von (C.6) hinzugefügt.
Danke für die Updates! Ich glaube, ich vermisse hier etwas. Sagen sie nicht am Anfang von Abschnitt 2.2 (Seite 7), dass die 2 N = 2 chiralen Multipletts innerhalb des N = 3-Hypermultiplets in konjugierten Darstellungen der Eichgruppe sind? Dann ist es falsch zu glauben, dass diese 2 N=2 chiralen Komponenten drin sind A D J = A D J ¯ der Spurweite Gruppe? Wenn die N F Hypermultiples sind im Grundton dann was ist die Darstellung in der die 2 N = 2 chirale Multipletts fallen in? Es wird großartig sein, wenn Sie diesen Punkt klären können!
Wie sie sagen, in der N = 3 Theoretisch können Sie Hypermultiplets haben, und jedes Hypermultiplet hat zwei chirale Multipletts: eines in einer Wiederholung R und eins drin R ¯ . Das ist nicht nötig R ist der Adjonnt. Etwas unterhalb der von Ihnen erwähnten Stelle erwägen sie verschiedene Möglichkeiten R , von denen nur eine adjungiert ist. Wenn Sie sagen, dass ein Hypermultiplet „fundamental“ ist (was meiner Meinung nach keine genaue Aussage ist, obwohl seine Bedeutung klar ist), bedeutet dies, dass Sie ein chirales Multiplett im Fundamental und eines im Antifundamental haben.
Danke für die Erklärung! Können Sie mir eine Referenz geben, wo diese masselosen Multipletts von N = 2 Und N = 3 Supersymmetrie ein 2 + 1 Abmessungen werden von Grund auf neu abgeleitet? Wenn Sie mir bei dieser vorherigen Frage helfen könnten, physical.stackexchange.com/questions/33023/… Das wäre eine große Hilfe!
Ich habe keine Referenz, aber die Geschichte ähnelt den Multiplets in D = 4 , die in jedem Lehrbuch zu SUSY hergeleitet werden. Sie können diese Übung in wiederholen D = 3 .
Ich bin nicht sicher. Bei d = 4 passiert das Besondere, dass die Antikommutierung zwischen den Qs diagonal ist und daher die Hälfte davon auf 0 geht und die andere Hälfte wie die Clifford-Algebra funktioniert. In d=3 existiert diese Vereinfachung wegen der Besonderheiten der Gamma-Matrizen bei dieser Dimension nicht. Daher würde ich gerne wissen, ob es eine Standardreferenz gibt, die die Multiplettkonstruktion in d = 3 erklärt.
Ich sagte, es ist ähnlich, nicht dasselbe. Mir scheint, dass diese Diskussion von der ursprünglichen Frage abgekommen ist. Warum nicht als neue Frage posten?
Ich hatte bereits die separate Frage gestellt, die von meinem vorherigen Kommentar verlinkt wurde. Sie ist unbeantwortet geblieben. Übrigens, ich habe dir das Kopfgeld zugesprochen. Danke für die Hilfe! Ich hoffe, Sie auch bei dieser anderen Frage von mir zu sehen!
Ich habe eine etwas bearbeitetere Version dieser Frage veröffentlicht, physical.stackexchange.com/questions/33023/… Es wäre großartig, Ihre Hilfe zu haben.
Ich denke, meine vorherige Frage war nicht richtig formuliert und ich habe hier hoffentlich eine bessere Aussage gemacht, physical.stackexchange.com/questions/34192/… Würde mich freuen, Sie dort zu sehen!
@GuyGur-Ari Einige kürzliche Gedanken brachten mich zurück zu dieser Diskussion mit Ihnen. Durch deine Konstruktion ρ N ist real - und daher, wenn sie von 5.6 auf 5.10 in der Veröffentlichung arxiv.org/pdf/hep-th/0310285v6.pdf gehen , hätten sie eine Quadratwurzel bekommen sollen - aber sie tun es nicht - es scheint, dass ihr \rho_n ist komplex - können Sie diesen Punkt bitte erläutern?